krok 2 - 2014 question paper (stomatology)

31
Кrok 2 Stomatology (англомовнi студенти) 2014 рiк 1 1. After examination a 5-year-old child was diagnosed with acute general pulpitis of the 74 tooth. What treatment will be most efcient in this case? A. Pulp extirpation B. Vital amputation C. Non-vital amputation D. Biological method E. Tooth extraction 2. A 38-year-old female patient complains of pain in the 16 tooth, which appeared 3 days after placing an arti- cial crown over the tooth. Objectively: the 16 tooth is crowned with a full metal swaged crown. The gingiva around the tooth is hyperemic, edematous. The crown margin is submerged into gingi- val pocket by more than 0,5 mm. What is the cause of this complication? A. Excessive submergence of the crown margin into the gingival pocket B. Excessively tight t of the crown C. Interdental contacts D. Contact of the crown with antagoni- sts E. Eating solid food 3. A 34-year-old female patient presented to a dental clinic for the prosthetics of the mandibular teeth. Objectively: the mandibular teeth exhi- bit the I-II class mobility. Which of the following examination methods should be applied? A. Radiography B. Galvanometry C. Face-bow record D. Rheotachygraphy E. Myotonometry 4. A 29-year-old female patient presented to a dental clinic for prosthetics. Objectively: there is a cari- ous cavity on the masticatory and mesi- al surfaces of the 24 tooth. Interdental contact is inadequate. Classify this cavi- ty according to Black’s classication: A. II B. I C. IV D. III E. V 5. A patient complains of spontaneous, paroxysmal, irradiating pain with short pain-free intervals. The pain arose 2 days ago and occurs only at night. Make a provisional diagnosis: A. Acute diffuse pulpitis B. Acute deep caries C. Exacerbation of chronic periodontitis D. Acute circumscribed pulpitis E. Acute purulent pulpitis 6. A 40-year-old female patient complains of swelling of the left submandibular region which appeared three days ago; elevation of body temperature up to 37, 4 o C . Objectively: the left submandibular region exhibits a soft tissue edema, the skin color is not changed. Palpation reveals a roundi- sh dense elastic node with a smooth surface sized 2,0 x1,5 cm. The node is mobile, somewhat tender. Mouth opening is not restricted, oral mucosa exhibits no visible changes, the crown of the 36 tooth is decayed by 2/3, its percussion causes pain response. What is the most likely diagnosis? A. Acute odontogenic serous lymphadenitis of the submandibular region B. Odontogenic abscess of the submandibular region C. Odontogenic phlegmon of the submandibular region D. Chronic odontogenic lymphadenitis of the submandibular region E. Acute serous sialadenitis of the submandibular salivary gland 7. A 70-year-old female patient complains of burning sensation under the base of her complete denture; dryness and bitterness in the mouth. The denture was fabricated 3 weeks ago. Objectively: the alveolar bone mucosa is edematous, there is diffuse hyperemia up to the margins of the denture. The tongue is hyperemic, dry and cracked. What additional study would be most appropriate in this case? A. Allergotest B. Urinalysis C. Complete blood count D. Biochemical blood analysis E. Sialosemiology 8. A 44-year-old female patient

Upload: eneutron

Post on 11-Jan-2017

445 views

Category:

Health & Medicine


14 download

TRANSCRIPT

Page 1: Krok 2 - 2014 Question Paper (Stomatology)

Кrok 2 Stomatology (англомовнi студенти) 2014 рiк 1

1. After examination a 5-year-oldchild was diagnosed with acute generalpulpitis of the 74 tooth. What treatmentwill be most efficient in this case?

A. Pulp extirpationB. Vital amputationC. Non-vital amputationD. Biological methodE. Tooth extraction

2. A 38-year-old female patientcomplains of pain in the 16 tooth, whichappeared 3 days after placing an artifi-cial crown over the tooth. Objectively:the 16 tooth is crowned with a full metalswaged crown. The gingiva around thetooth is hyperemic, edematous. Thecrown margin is submerged into gingi-val pocket by more than 0,5 mm. Whatis the cause of this complication?

A. Excessive submergence of the crownmargin into the gingival pocketB. Excessively tight fit of the crownC. Interdental contactsD. Contact of the crown with antagoni-stsE. Eating solid food

3. A 34-year-old female patientpresented to a dental clinic for theprosthetics of the mandibular teeth.Objectively: the mandibular teeth exhi-bit the I-II class mobility. Which of thefollowing examination methods shouldbe applied?

A. RadiographyB. GalvanometryC. Face-bow recordD. RheotachygraphyE. Myotonometry

4. A 29-year-old female patientpresented to a dental clinic forprosthetics. Objectively: there is a cari-ous cavity on the masticatory and mesi-al surfaces of the 24 tooth. Interdentalcontact is inadequate. Classify this cavi-ty according to Black’s classification:

A. IIB. IC. IVD. IIIE. V

5. A patient complains of spontaneous,

paroxysmal, irradiating pain with shortpain-free intervals. The pain arose 2days ago and occurs only at night.Make a provisional diagnosis:

A. Acute diffuse pulpitisB. Acute deep cariesC. Exacerbation of chronic periodontitisD. Acute circumscribed pulpitisE. Acute purulent pulpitis

6. A 40-year-old female patientcomplains of swelling of the leftsubmandibular region which appearedthree days ago; elevation of bodytemperature up to 37, 4oC. Objectively:the left submandibular region exhibitsa soft tissue edema, the skin color is notchanged. Palpation reveals a roundi-sh dense elastic node with a smoothsurface sized 2,0 x1,5 cm. The nodeis mobile, somewhat tender. Mouthopening is not restricted, oral mucosaexhibits no visible changes, the crownof the 36 tooth is decayed by 2/3, itspercussion causes pain response. Whatis the most likely diagnosis?

A. Acute odontogenic serouslymphadenitis of the submandibularregionB. Odontogenic abscess of thesubmandibular regionC. Odontogenic phlegmon of thesubmandibular regionD. Chronic odontogenic lymphadenitisof the submandibular regionE. Acute serous sialadenitis of thesubmandibular salivary gland

7. A 70-year-old female patientcomplains of burning sensation underthe base of her complete denture;dryness and bitterness in the mouth.The denture was fabricated 3 weeksago. Objectively: the alveolar bonemucosa is edematous, there is diffusehyperemia up to the margins of thedenture. The tongue is hyperemic, dryand cracked. What additional studywould be most appropriate in this case?

A. AllergotestB. UrinalysisC. Complete blood countD. Biochemical blood analysisE. Sialosemiology

8. A 44-year-old female patient

Page 2: Krok 2 - 2014 Question Paper (Stomatology)

Кrok 2 Stomatology (англомовнi студенти) 2014 рiк 2

presented to a dental clinic forprosthetics. Objectively: the 17, 16, 15,14, 12, 25, 26 teeth are missing; the18, 28 teeth are preserved. Make a di-agnosis according to Kennedy classifi-cation of partial edentulousness:

A. Class III, subclass 2B. Class III, subclass 1C. Class III, subclass 3D. Class II, subclass 2E. Class II, subclass 4

9. A 45-year-old female patientpresents to a dental clinic for prostheti-cs. Objectively: the 47, 46, 45, 35,36, 37 teeth are missing. There isdentoalveolar vertical displacement ofthe 17, 16, 26, 27 teeth; the alveolarprocess is hypertrophied, the cervicalmargins are not exposed. When theteeth are closed, the distance betweenthe occlusal surfaces of teeth and thealveolar process is about 6 mm. Specifythe most efficient method of treatmentof secondary deformation in this pati-ent:

A. Pulp removal and teeth grindingB. Surgical treatmentC. GrindingD. Hardware-associated surgicaltreatmentE. Disocclusion

10. A 25-year-old female patientconsulted a dentist about acute painin the mandible on the right that occursduring eating. Objectively: the approxi-mate distal surface of the 45 toothexhibits a carious cavity filled with li-ght softened dentin. Probing causes aminor pain response across the entirefloor. Percussion causes no pain. Coldwater causes transient pain. What is themost likely diagnosis?

A. Acute deep cariesB. Acute median cariesC. Chronic deep cariesD. Chronic median cariesE. Chronic fibrous pulpitis

11. A 45-year-old male patient hasbeen admitted to the maxillofacialdepartment with complaints of generalweakness, pain and swelling in theleft submandibular region, elevationof body temperature up to 39oC. He

has a history of frequent pain in the 36mandibular tooth on the left. The faceis asymmetric due to the swelling ofthe left submandibular region, where adense infiltrate is palpable; the overlyi-ng skin is hyperemic, doesn’t make afold. There is a deep carious cavityin the 36 tooth, its percussion causespain response; the surrounding mucosais hyperemic, edematous. What is themost likely diagnosis?

A. Phlegmon of the left submandibularregionB. Acute left-sided submandibularsialadenitisC. Acute suppurative left-sided periosti-tis of the mandibleD. Acute suppurative left-sidedsubmandibular lymphadenitisE. Acute left-sided osteomyelitis of themandible

12. A month after cementing aporcelain-fused-to-metal crown for the23 tooth, the patient complained of itsdecementation. On examination thetooth stump was of sufficient height,the stump walls were convergent to thevertical axis at an angle of about 30degrees. What is the required angle ofwall convergence?

A. Up to 8 degreesB. 18-20 degreesC. 20-25 degreesD. 25-30 degreesE. 27-35 degrees

13. A 5-year-old child has been di-agnosed with chronic granulating peri-odontitis of the 85 tooth. What is theoptimal material for the root canal filli-ng?

A. Zinc-eugenol cementB. Zinc-phosphate cementC. Glass ionomer cementD. Calcium-containing pasteE. Resorcinol-formalin paste

14. A 65-year-old male patientundergoes fabrication of a fullremovable denture for the mandible.The individual tray for taking a functi-onal impression is adjusted with the useof Herbst tests. The tray is thrown offduring swallowing. This means that itsedge is to be shortened in the following

Page 3: Krok 2 - 2014 Question Paper (Stomatology)

Кrok 2 Stomatology (англомовнi студенти) 2014 рiк 3

area:

A. From the space behind the mucoustubercle to the mylohyoid lineB. From canine to canine vestibularlyC. From canine to canine linguallyD. Along the mylohyoid lineE. In the region of the premolars on thelingual side

15. A 28-year-old female patientcomplains of a cosmetic defect inthe anterior segment of the maxilla.Objectively: the crown of the 13 toothis decayed nearly up to the gingivaledge, the stump walls are of suffici-ent thickness. The stump has a sli-ghtly changed color, percussion causesno pain response. Radiographical fi-ndings: the root canal is straight, wi-de, and filled along the entire length.What prosthetic construction is mostappropriate in this case?

A. Post and coreB. Plastic crownC. Porcelain-fused-to-metal crownD. InlayE. Micro-denture

16. A 32-year-old patient complains ofconstant chewing-like movements ofthe mandible. These movements occurwith nervous excitement. The patient’swife reports him to have gnashing ofteeth at night. Objectively: the lateralteeth are worn, have no tubercles.What is the optimal treatment tactics?

A. Night guard (Michigan-type splint),myogymnastics, self-massageB. Restoration with crownsC. Pharmaceutical and non-invasivetreatment following the referral to adental surgeonD. Referral to a dental surgeonE. Alignment of teeth (selective grindi-ng)

17. A 38-year-old female patientcomplains of pain in front of theexternal auditory meatus projection,clicking when opening the mouth,sensation of stuffiness in the ears.Objectively: the face is symmetric, thetrajectory of mouth opening is straight.There is a I class Kennedy defect, the18, 17, 16, 26, 27, 28 teeth are missing.

Which anatomical structure takes thegreatest load in this situation?

A. Articular disc (meniscus)B. Articular headC. Distal surface of the medial articulartubercleD. Bottom of the glenoid fossa of thetemporal boneE. Articular capsule

18. A 70-year-old male patient with thecompletely edentulous maxilla and ahard palate defect complains of difficulteating, pouring of liquid food throughthe nose, frequent chronic diseasesof the nasal mucosa. Objectively: themaxillary alveolar bone is atrophied(Schroeder class I), the mucosa pli-ability complies with Supple class I.The hard palate exhibits a cicatricallychanged defect sized 2x3 cm. Whatdenture design would be most effectivein this case?

A. Complete removable denture withan internal and a peripheral valveB. Complete removable denture withthe teeth placed on the artificial gingivaC. Soft-core obturator and completeremovable dentureD. Complete removable denture with athickening in the defect regionE. Complete removable denture with ametal base and soft-core obturator

19. A 7-year-old child complainsof fever up to 38, 3oC, headache,sore throat and general weakness.Objectively: the mucosa of the softpalate, tonsils and palatine archesis edematous and hyperemic, thereare numerous small painful erosi-ons with red bottom. The erosionsare not covered with any plaque.The submandibular lymph nodes areenlarged, painful on palpation. What isthe most likely diagnosis?

A. Herpetic anginaB. Acute herpetic stomatitisC. Infectious mononucleosisD. Scarlet fever stomatitisE. Diphtheric stomatitis

20. A 22-year-old patient complains ofa painful swelling in the right parotidgland region. A week before the pati-

Page 4: Krok 2 - 2014 Question Paper (Stomatology)

Кrok 2 Stomatology (англомовнi студенти) 2014 рiк 4

ent got a cheek abrasion which washealing under the purulent crust. Overthe past two days, the patient hadhad progressing pain and fever up to38, 6oC. Objectively: there is a soft ti-ssue edema in the right parotid region,the skin is slightly tense but has notchanged in colour. There is a dense pai-nful infiltration 2,5x3,5 cm large, theoverlying skin exhibits limited mobili-ty. The mouth can be fully opened, themucous membrane around the orificeof the salivary duct is unchanged, thesaliva is transparent. What is the mostlikely diagnosis?

A. Acute lymphadenitisB. Exacerbation of chronic parotitisC. Abscess of the parotid-massetericregionD. Acute non-epidemic parotitisE. Epidemic parotitis

21. A 42-year-old female pati-ent consulted a dentist about asudden dramatic enlargement of gi-ngiva, bleeding sores on the mucousmembrane, lymph node enlargement.There is body temperature elevation upto 38oC, weakness, pain in the bones.What disease can be suspected in thispatient?

A. Acute leukemiaB. Chronic leukemiaC. Hypovitaminosis CD. Hodgkin’s lymphomaE. Eosinophilic granuloma

22. A 56-year-old male patientpresents to a dental clinic for complexdental treatment prior to prostheticprocedures. Objectively: there is apathological grayish-white triangularlesion sized 0,5x1,5 cm on the buccalmucosa in the mouth corners. The lesi-on cannot be removed. There are nosigns of inflammation, regional lymphnodes are unchanged. Make a di-agnosis:

A. Planar leukoplakiaB. Mild leukoplakiaC. Typical lichen planusD. Mycotic stomatitisE. Hyperkeratotic lichen planus

23. A 24-year-old female patientpresented to a dental clinic for the

extraction of the maxillary wisdomtooth root. Following the posteri-or superior alveolar nerve block, apost-injection hematoma occurred.What structure was damaged duringanesthesia?

A. Pterygoid venous plexusB. Maxillary arteryC. Infraorbital arteryD. Zygomatic arteryE. Palatine artery

24. What is the most adequate surgi-cal approach for a submandibularphlegmon dissection?

A. 5-6 cm long incision 2 cm below themandibleB. 5-6 cm long incision bypassing theangle of the mandibleC. Intraoral approach along the mucogi-ngival junction at the molar levelD. Linear 2 cm long incision along thelower edge of the mandibleE. 5-6 cm long incision along the skincrease of the upper cervical region

25. A 12-year-old patient has chroniccatarrhal gingivitis. The oral vestibuleis 7 mm deep. Fedorov-Volodkina hygi-ene index is 4. Examination revealedno abnormalities of lip frenulumattachment. What measures should betaken in the first place?

A. Professional oral hygieneB. VestibuloplastyC. Curettage of periodontal pocketsD. PhysiotherapyE. -

26. On the basis of subjective complai-nts, anamnesis and objective exami-nation, a 5-year-old child has beendiagnosed with impacted dislocationof the 71 tooth. What is the mostappropriate treatment tactics?

A. Extraction of the 71 toothB. Reposition and fixation of the 71toothC. Orthodontic treatment of the 71toothD. Endodontic treatment of the 71toothE. No intervention is required

27. A 26-year-old male patient presents

Page 5: Krok 2 - 2014 Question Paper (Stomatology)

Кrok 2 Stomatology (англомовнi студенти) 2014 рiк 5

to a prosthetic dentistry clinic for aswaged metal crown for the 26 tooth.This is his third visit. During examinati-on he has been found to have a hole onthe medial buccal tubercle of the crownwhich appeared as a result of polishing.What tactics should be chosen?

A. Fabricate a new crownB. Seal up the hole in the crownC. Cement the crown with the cementD. Mount a fixed dentureE. Cement the crown with acryl oxide

28. A 28-year-old patient complainsthat a piece of the 23 tooth crownhas broken off. Objectively: the rootcanal orifices contain the filling materi-al, percussion causes no pain response.What is the tactics of choice?

A. Radiography of the 23 toothB. Extraction of the 23 toothC. Fabrication of a metal insert toothD. Ortho-polymer restoration of the 23toothE. Restoration with Belkin crown

29. A 58-year-old patient complains ofstuffiness in the ears, hearing impai-rment, pain in the parotid region whenopening the mouth. Objectively: the 18,17, 16, 15, 26, 27, 28 teeth are restoredwith swaged metal crowns. There is a1,5-2 mm gap between the crown edgesand gingiva. The 14 and 25 teeth exhi-bit carious cavities. Probing, percussi-on and thermal stimuli cause no painresponse. The patient has deep incisoroverbite. What is the likely cause ofotologic symptoms?

A. Functional TMJ overloadB. Complication of tooth decay in the14, 25 teethC. Missing mandibular teethD. Cervical margin exposure of the 18,17, 16, 15, 26, 27, 28 teethE. Galvanosis

30. A 32-year-old patient presentedto a prosthetic dentistry clinic witha diagnosis of maxillary fracture.Thefracture can be treated by means ofstandard maxillo-mandibular Zbarzhset. What construction provides theintra-oral fixation?

A. Standard double archB. Aluminium arch barC. Plastic biteplateD. Weber’s splintE. Crown-supported soldered splint

31. A 25-year-old male complains ofpain, a sensation of heaviness in themaxilla on the left. He has a historyof the 25 tooth extraction followed byhealth deterioration. There appearedpurulent discharge from the left nasalmeatus. He was diagnosed with acuteodontogenic left-sided sinusitis. Whichnasal meatus should the maxillary sinusbe punctured through?

A. InferiorB. SuperiorC. MiddleD. Superior and inferiorE. Middle and inferior

32. A 22-year-old male patientpresented to a dental clinic with acavity in the 11 tooth. The pati-ent had no previous history of thistooth treatment. Objective examinati-on revealed a carious cavity filled withlight softened dentin on the approxi-mate medial surface. The carious cavitywas located within the mantle dentin.The tooth is slightly sensitive to thecold stimuli. What is the most likelydiagnosis?

A. Acute median cariesB. Acute deep cariesC. Chronic median cariesD. Chronic deep cariesE. Superficial caries

33. A patient consulted an oral surgeonabout maxillary pain on the left, whi-ch arose three days ago. After exami-nation, the patient was diagnosed wi-th exacerbation of chronic periodonti-tis of the 17 tooth. The 17 tooth isindicated for extraction. What nervesshould be blocked for a painlessextraction of the 17 tooth?

Page 6: Krok 2 - 2014 Question Paper (Stomatology)

Кrok 2 Stomatology (англомовнi студенти) 2014 рiк 6

A. Posterior alveolar nerves and greaterpalatine nerveB. Greater palatine nerveC. Anterior alveolar nerves and incisivenerveD. Middle alveolar nerves and greaterpalatine nerveE. Nasopalatine nerve

34. A 35-year-old patient complainsabout itch, burning and edema of li-ps. These presentations occured a weekago. Objectively: there is reddening ofvermilion border and lip skin, especi-ally in the region of mouth corners,there are also vesicles, crusts, smallcracks accompanied by erythematouslesion of vermilion border. What is themost likely diagnosis?

A. Acute eczematous cheilitisB. Exudative erythema multiformeC. Acute herpetic cheilitisD. Allergic contact cheilitisE. Exudative form of exfoliative cheilitis

35. A 6-year-old child complains ofpain in a mandibular tooth on the leftduring eating. Objectively: masticatorysurface of the 36 tooth exhibits a cari-ous cavity within the mantle dentin.The cavity is full of light softeneddentin which can be easily removed wi-th an excavator. Probing of the cavitywalls causes pain response. The teethare sensitive to thermal stimuli, thepain is of short-term nature. There isno response to percussion. Select theoptimal filling material:

A. Silver amalgamB. Zinc phosphate cementC. Silicate cementD. Silicophosphate cementE. Polycarboxylate cement

36. A patient undergoes orthopedictreatment for bounded edentulousspaces in the upper jaw with fixed full-cast dentures. At the second appoi-ntment it is necessary to check whetherthe inner surface of the metal frameof the future porcelain-fused-to-metalbridge fits the surface of the preparedteeth. How can this be done?

A. In the oral cavity by means of sili-cone materialsB. By means of visual models in thearticulatorC. In the oral cavity by means of transferpaperD. In the oral cavity by means of a waxplateE. In the oral cavity by means ofstomatoscopy

37. A 44-year-old male patient needs acast splint for the mandible. Objecti-vely: there is a bilateral terminal defectof the mandibular dental arch. Thetooth crowns are high, the teeth areintact, there is I-II class mobility. Whatimpression material should be used?

A. StomalginB. RepinC. GypsumD. StensE. Dentafol

38. A 35-year-old male complains ofshort-term pain caused by thermal sti-muli in the 46 and 47 teeth. Objecti-vely: masticatory surfaces of the 46, 47teeth are intact, approximate surfacescould not be examined because of avery close arrangement of teeth. Whatmethods of study can be used in thiscase if you suspect the presence of hi-dden cavities?

A. X-rayB. Vital stainingC. Fluorescent studyD. Measuring electrical resistanceE. Electro-odontometry

39. A 13-year-old child complains ofhaving a cavity in the front maxillaryteeth. Contact medial surfaces of the11 and 21 teeth exhibit cavities foundwithin the mantle dentine and filledwith dense pigmented dentin. Probi-ng of the cavity floor causes no painresponse, neither does dental percussi-on. Select the best filling material forthe permanent seals:

A. Resin compositeB. Silicate cementC. Silicophosphate cementD. Zinc phosphate cementE. Glass ionomer cement

Page 7: Krok 2 - 2014 Question Paper (Stomatology)

Кrok 2 Stomatology (англомовнi студенти) 2014 рiк 7

40. Parents of a 3-year-old child reportthat the child has persistent pain inthe front maxillary teeth. Objecti-vely: the coronal part of the 61 toothis gray and decayed. Probing of theroot canal orifice causes pain responseand is accompanied by bleeding. Thetooth percussion provokes acute pain.Mucosa is hyperemic, edematous andtender. Palpation in the region of the61, 62 teeth reveals a fistula. What isyour provisional diagnosis?

A. Exacerbation of chronic periodonti-tisB. Acute purulent periodontitisC. Acute diffuse pulpitisD. Chronic granulating periodontitisE. Exacerbation of chronic pulpitis

41. A 15-year-old teenager consultedan orthodontist about misalignmentof teeth. Objectively: the faceis unremarkable. The patient haspermanent dentition. There are noabnormalities of jaw relationship inthree dimensions. The 23 tooth has avestibular position and is located abovethe occlusal plane, the space in thedental arch is less than 1/3 of crown.The space for the misaligned 23 toothcan be gained due to:

A. Extraction of the 24 toothB. Increase of the transverse jaw di-mensionsC. Increase of the sagittal jaw dimensi-onsD. Extraction of the 23 toothE. Increase of the vertical jaw dimensi-ons

42. An 11-year-old boy complains of acarious cavity in a mandibular toothon the right. Objectively: the 46 toothexhibits a carious cavity within themantle dentin. The dentin is dense, pi-gmented; there is pain response to thecold stimulus, probing and percussioncause no pain response. Make a provi-sional diagnosis:

A. Chronic median cariesB. Acute median cariesC. Acute deep cariesD. Chronic deep cariesE. Chronic superficial caries

43. A 45-year-old male patient complai-

ns of inertness, headache, fever upto 39oC, burning neuralgic pain inthe mandible on the right. Objecti-vely: the skin of chin on the right ishyperemic, there is vesicular rash wi-th a vesicle diameter of 1 to 6 mm.Similar vesicles can be found on thehyperemic oral mucosa on the right.They burst easily making erosions whi-ch are covered with fibrinous pelli-cles. Regional lymphadenitis is present.What is the most likely diagnosis?

A. ShinglesB. Acute herpetic stomatitisC. Allergic stomatitisD. PemphigusE. Exudative erythema multiforme

44. A 50-year-old patient complai-ns of dryness and burning sensati-on in the mouth, pain during eati-ng, taste perversion. The symptomsoccurred a week ago after taking thecourse of antibiotics for acute bronchi-tis. Objectively: oral mucosa is coveredwith patches of white caseous deposit.The deposit can be easily removedwhen scraped, revealing hyperemicareas of mucous membrane. Which isthe most likely diagnosis?

A. Acute pseudomembranous candidi-asisB. Chronic hyperplastic candidiasisC. LeukoplakiaD. Lichen planusE. Chronic recurrent aphthous stomati-tis

45. A 56-year-old female patientcomplains of burning neuralgic painin the left half of the mandible, whi-ch appeared two days ago. There is alsorash in form of tense vesicles on thereddened skin and mucosa of the lowerlip on the left, accompanied by malaise,headache, body temperature elevationup to 38oC, left-sided lymphadenitis.What is the most likely diagnosis?

A. Herpes zosterB. MurrainC. Acute herpetic stomatitisD. Allergic reactionE. Pemphigus

46. A 21-year-old dental patient hasbeen found to have a circular ulcer on

Page 8: Krok 2 - 2014 Question Paper (Stomatology)

Кrok 2 Stomatology (англомовнi студенти) 2014 рiк 8

the tongue dorsum. The ulcer is up to1 cm in diameter, has demarcated rai-sed edges and a deep inflammatory infi-ltrate at the base. The ulcer floor is flat,clean, of crimson color. The regionallymph nodes are of cartilaginous densi-ty, mobile, painless. The ulcer appeareda week ago and doesn’t cause any di-scomfort. What kind of ulcer should besuspected in the first place?

A. SyphiliticB. TrophicC. TraumaticD. TuberculousE. Cancerous

47. An 11-year-old boy complains ofa short-term pain from the cold in aleft mandibular tooth. Objectively: themedial surface of the 36 tooth exhibi-ts a carious cavity within parapulpardentin. The cavity is filled with li-ght, softened dentin and does notcommunicate with the cavity of thetooth. Probing the of the 36 tooth floorcauses pain response, the tooth is notsensitive to percussion, the response tothe cold stimulus does not stay longafter its removal. What is the most li-kely diagnosis?

A. Acute deep cariesB. Acute focal pulpitisC. Acute diffuse pulpitisD. Chronic fibrous pulpitisE. Acute median caries

48. A 36-year-old patient with the faceand neck burns resulting from gasoli-ne ignition has been delivered to theemergency room. Palpebral fissures areclosed due to the eyelid edema. Theface skin is erythematous, there arelarge tense blisters on the skin of thechin, cheeks, nose, forehead, superiorsurface of neck. Specify the degree ofburns:

A. II degreeB. I degreeC. III degreeD. IV degreeE. -

49. A 9-year-old child has a symmetri-cal maxillary diastema with crowndivergence. The relationship of thelateral teeth is neutral. In the anterior

segment the depth of incisal overbite is1/3 of the crown height. What appliancecan be used for simultaneous treatmentof diastema and crown divergence?

A. Maxillary plate with spring armsB. Maxillary plate with protractionspringsC. Standard edgewise techniqueD. Bracket system with Andrew’sstraight-wireE. Maxillary plate with vestibular arch

50. A 52-year-old female patientcomplains of a persistent burningsensation at the tip and lateral surfaceof the tongue. The burning occursin the morning and gets worse inthe evening, the sensation disappearswhen eating. The patient reports havi-ng had these symptoms for about ayear and associates the disease withwearing removable dentures. The pati-ent has a history of anacid gastritis,a surgery for uterine fibroids, seconddegree hypertension, poor sleep, anxi-ety. Objectively: the tongue is sli-ghtly swollen, the tongue dorsum iscovered with a thin white plaque, fi-liform papillae are atrophied. Therest of the mucosa exhibits no visi-ble pathological changes. Pharyngealreflex is suppressed. What is the mostlikely diagnosis?

A. GlossalgiaB. Allergic reaction to plasticC. Moeller-Hunter glossitisD. Candidal glossitisE. Lingual nerve neuritis

51. A 28-year-old female patientconsulted a dentist about pain causedby thermal and chemical stimuli inthe mandibular and maxillary teeth.Objectively: in the cervical zone ofmandibular and maxillary incisors andcanines there are irregularly shapeddefects of hard dental tissues withinthe amelodentinal junction in form oftissue softening. In this clinical case adentist will find it difficult to differenti-ate between the following diseases:

Page 9: Krok 2 - 2014 Question Paper (Stomatology)

Кrok 2 Stomatology (англомовнi студенти) 2014 рiк 9

A. Caries and necrosisB. Hypoplasia and necrosisC. Hypoplasia and wedge-shaped defectD. Caries and erosionE. Erosion and wedge-shaped defect

52. A 52-year-old male patient worki-ng as a driver consulted a dentist aboutpain, difficult mouth opening. The pati-ent had been diagnosed with ulcerativeleukoplakia. After a 10-day course oftherapy sluggish granulation began. Onthe periphery of the ulcer the keratini-zation processes became more active.Which of the following tests must beconducted in the first place?

A. MicroscopyB. BacterioscopyC. StomatoscopyD. Bacteriological stdyE. Cytological study

53. 4 hours after application of arsenicpaste to the 36 tooth pulp, a 27-year-old patient complained of pain in thetreated tooth. Objectively: on the distalsurface of the 36 tooth the temporaryfilling is present. Percussion causes nopain response. Gingival papilla in theregion of the 36, 37 teeth is hyperemicand edematous, its apex is necrotized.What is the optimal tactics of this pati-ent management?

A. Replace the temporary filling, treatthe gingiva with 3% iodine solutionB. Keep the temporary filling, admini-ster soda rinseC. Keep the temporary filling, treat thegingiva with 3% iodine solutionD. Replace the temporary filling, treatthe gingiva with 3% hydrogen peroxidesolutionE. Keep the temporary filling, treat thegingiva with 3% hydrogen peroxidesolution

54. A 43-year-old female patient hasbeen diagnosed with generalized peri-odontitis. What examination is mostinformative for determining the severi-ty grade of the disease?

A. Panoramic radiographyB. Schiller-Pisarev testC. Measuring the depth of periodontalpocketsD. Calculation of the periodontal indexE. Determining the class of tooth mobi-lity

55. A 7,5-year-old boy fell down onhis face and damaged his front maxi-llary teeth. Objectively: the crowns ofthe 11 and 12 teeth are destroyed by1/4. The tooth cavity is closed. Probi-ng causes pain along the break-off line,percussion of the 11 and 12 teeth causesno pain response. Mucosa around the11 and 12 teeth is slightly hyperemic.Radiograph of the 11 and 12 teethshows the intactness of the dental ti-ssues and alveolar process. What tacti-cs of a dentist would be optimal in thefirst hours after the injury?

A. Pulp monitoringB. Fabrication of an artificial crownC. Photopolymer fillingD. Parapulpar pin reconstructionE. Composite filling

56. A 9-year-old child complains ofpain in the left mandibular molarthat occurs during eating. The masti-catory surface of the 75 tooth exhibi-ts a carious cavitiy filled with softeneddentin and localized within parapulpardentin. The cavity communicates wi-th the tooth cavity. Probing at the si-te of communication causes acute painand moderate bleeding. Percussion ofthe tooth causes no pain response.Radiography revaled no periodontalalterations in the 75 tooth. Select themethod of treatment:

A. Devital amputationB. Devital extirpationC. Vital extirpationD. Vital amputationE. Biological method

57. Preventive examination a 6-year-oldchild revealed that the child had deci-duous dentition, direct incisor contactin the frontal segment, no gaps betweenteeth; contact of homonymous caninesand molars; abrasion of masticatorytubercles of the molars. The child’scondition corresponds with the followi-

Page 10: Krok 2 - 2014 Question Paper (Stomatology)

Кrok 2 Stomatology (англомовнi студенти) 2014 рiк 10

ng period:

A. Physiological wear of deciduousdentitionB. Development of deciduous dentitionC. Stable deciduous dentitionD. Mixed dentitionE. Exfoliation

58. A 12-year-old girl visited a dentistfor a regular check-up. Objectively: theteeth are intact, the bite is open, themucosa in the papillary and marginalregions is hyperemic and cyanotic, thegingival papillae are roundish, enlargedand cover one third of the crown heightof the anterior maxillary teeth. Make aprovisional diagnosis:

A. Hypertrophic gingivitisB. Chronic catarrhal gingivitisC. Acute catarrhal gingivitisD. Gingival fibromatosisE. -

59. A 29-year-old male patient complai-ns of gingival hemorrhages duri-ng tooth brushing, and bad breath.Objectively: gingival papillae andmarginal gingiva are hyperemic witha cyanotic tint, bleed on probing. Peri-odontal pockets are 3-3,5 mm deep, wi-th serous exudate expressed from them.The sub- and supragingival calculusis present. Orthopantomograph showsosteoporosis of the alveolar process,resorption of interalveolar septa to 1/3.What is the most likely diagnosis?

A. Chronic generalized I grade peri-odontitisB. Exacerbation of chronic generalizedI grade periodontitisC. Chronic generalized periodontitis,initial stageD. Exacerbation of chronic generalizedperiodontitis, initial stageE. Chronic generalized II grade peri-odontitis

60. A 40-year-old patient complainsof fever up to 38oC, and a roundishinfiltration on the upper lip. Objecti-vely: the upper lip on the left exhibi-ts a round infiltrate, the overlying skinis deep crimson. The infiltrate adheresto the surrounding tissues and has anecrotic core in the center. The upperlip is hyperemic, edematous. What di-

agnosis can be made?

A. Upper lip furuncleB. Acute abscess of the upper lipC. Retention cystD. Acute lymphadenitisE. Upper lip carbuncle

61. A 20-year-old female patientconsulted a dentist about a short-termpain in the 22 tooth caused by cold sti-muli or contact with food. The painhad been observed for several months.After the examination, the patient wasdiagnosed with acute deep caries of the22 tooth. What material should be usedas a base for the cavity filling?

A. CalcidontB. Paste with trypsinC. Phosphate cementD. Silver-containing phosphate cementE. Silidont

62. A 48-year-old female patientunderwent parotidectomy for a beni-gn tumor of the parotid gland.Postoperatively, the patient wasobserved to have mouth downturni-ng, midline deviation towards theunaffected side, speech disturbance.Which branch of the facial nerve wasdamaged during the surgical interventi-on?

A. Marginal mandibular branch offacial nerveB. Buccal branchC. Zygomatic branchD. Cervical branchE. Temporal branch

63. On examination a 42-year-oldpatient was diagnosed with acutesuppurative periostitis of the mandiblebeginning from the 35 tooth. What isthe most rational method of anesthesiafor the 35 tooth extraction and dissecti-on of the subperiosteal abscess?

A. Torusal anesthesiaB. Berchet anesthesiaC. Plexus anesthesiaD. Infiltration anesthesiaE. General anesthesia

64. Examination of a 31-year-old pati-ent revealed perforation of the pulpchamber floor of the 36 tooth with a

Page 11: Krok 2 - 2014 Question Paper (Stomatology)

Кrok 2 Stomatology (англомовнi студенти) 2014 рiк 11

minor radiolucency of the interradi-cular septum. Which of the followingmethods of surgical treatment shouldbe applied in order to save the 36 toothand use it as an abutment tooth for abridge denture?

A. Coronary radicular separationB. Root amputationC. Root hemisectionD. Tooth replantationE. Sealing of perforation

65. A 42-year-old patient complainsof pain in the right side of her head,restricted movements of the lowerjaw, clicking sound, periodic spasmsof chewing muscles. Objectively: theface is symmetric, mouth opening isrestricted. On palpation of the ri-ght temporo-mandibular joint (TMJ)there are crepitation and clicking thataccompany the mandible movements.Examination of the oral cavity revealedalso a Kennedy’s class II defect on theright. What is the most likely diagnosis?

A. Pain dysfunction of the right TMJB. Acute arthritis of the right TMJC. Sclerosing osteoarthritis of the rightTMJD. Contracture of the right TMJE. Myositis ossificans

66. A 40-year-old patient complai-ns of malaise, weakness, headache,fever up to 38, 5oC. Objectively: thereis a symmetric red lesion in form ofbutterfly-wing rash on the face aroundthe nose and cheeks. The lesion isclearly demarcated, tender on palpati-on, does not disappear when pressedupon. The skin is raised and glossy.Submandibular and parotid superfici-al lymph nodes are enlarged, tender topalpation. What is the most likely di-agnosis?

A. Facial erysipelasB. Systemic lupus erythematosusC. Furuncle of faceD. Microbial eczemaE. Neurodermatitis

67. A 43-year-old male patient awai-ts fabrication of full swaged crownsfor the 26 and 27 teeth. What materi-al should be used for the dental dies?

A. Low-fusible alloyB. Stainless steelC. Cobalt-chromium alloyD. Silver-palladium alloyE. Solder alloy

68. Examination of a 9-year-old chi-ld revealed chalky spots in the cervi-cal zone of the 12, 11, 21, 22 teeth.The spots appeared two weeks ago.Lustreless surface of the spots isstainable with methylene blue. Theaffected teeth are not sensitive to thecold stimulus. Specify the physician’stactics in respect of the affected teeth:

A. Remineralizing therapyB. Impregnation therapyC. Grinding of affected areasD. Case follow-upE. -

69. An 18-year-old male has a furunclein the region of the mandible angle.The surrounding tissues are rigid.Cyanosis and induration in form of acord extend to the corner of the eye.General condition is deteriorated, bodytemperature is of 38, 5oC. What compli-cation has developed in this case?

A. Facial vein thrombophlebitisB. Abscess of cheekC. Phlegmon of cheekD. Acute sinusitisE. Carbuncle of cheek

70. Regimental aid station admitted awounded soldier with a diagnosis ofa gunshot fracture of the mandibularbody. What kind of immobilization isrequired at this stage?

A. Transport immobilizationB. Immobilization with splints for bothjawsC. Extraoral appliance for fragmentfixationD. Smooth braceE. Osteosynthesis of the mandible

71. A 17-year-old male patient complai-ns about having a fistula on the anteriorsurface of his neck. The fistula has beenpresent since birth. Periodically thereappears a small amount of mucoiddischarge. The patient has been di-agnosed with a middle cervical fistula.

Page 12: Krok 2 - 2014 Question Paper (Stomatology)

Кrok 2 Stomatology (англомовнi студенти) 2014 рiк 12

What tactics of treatment should bepreferred for this patient?

A. Excision of the fistulous tract withresection of the hyoid bone bodyB. Excision of the fistulous tract withoutresection of the hyoid bone bodyC. Closure of the fistula with the purse-string sutureD. Cryodestruction of the fistula epi-theliumE. Sclerosing of the fistula epithelium

72. A 70-year-old patient complains ofinability to take food, a cosmetic andphonetic defect due to the completeloss of mandibular teeth. Objectively:the lateral segment of the alveolarprocess of the mandible is significantlyatrophied, while the frontal segment isrelatively preserved. Buccal folds areattached at the crest of the alveolarprocess. These clinical presentationscorrespond with the following class ofedentulous jaws according to Kellerclassification:

A. Class IIIB. Class IIC. Class ID. Class IVE. Class V

73. A 24-year-old male who had beeninjured in a fight was delivered to themaxillofacial department. The patientwas diagnosed with a fracture of the leftzygomatic bone with a displacement;a fracture of the anterior wall of themaxillary sinus. What kind of surgery isindicated for this patient?

A. Radical maxillary sinusotomy andreposition of fragmentsB. Radical maxillary sinusotomyC. Osteotomy of the maxillaD. Osteosynthesis of the zygomaticboneE. Reposition of fragments

74. A 50-year-old male patient has agunshot wound to the mandible and abone defect in the mental area. Whatmethod of immobilization is indicatedfor this patient?

A. Extraoral Rudko apparatusB. Two-jaw splint with wire loops andintermaxillary elasticsC. Bone sutureD. One-jaw braceE. Immobilization by means of wiresand rods

75. 3 days ago a 35-year-old patientunderwent the extraction of the 47tooth, which was followed by healthdeterioration. Objectively: the pati-ent is pale, body temperature is of38, 1oC, the mandible on the left is thi-ckened, the surrounding soft tissuesare swollen, mouth opening is difficult.Mucogingival junction at the 48, 47, 46teeth is edematous, mucous membraneis hyperemic. There are purulent di-scharges from the 47 tooth socket, the48, 46, 45 teeth are sensitive to percussi-on. There is also paresthesia in thelower lip region. What is the most li-kely diagnosis?

A. Acute mandibular osteomyelitisB. Acute mandibular periostitisC. Submandibular phlegmonD. Acute periodontitis of the 47 toothE. Abscess of mandibular radicular cyst

76. A female patient presented to aclinic for prosthetics. Objectively: thelower third of the face is shortened;nasolabial folds are deepened; the frontteeth of both jaws are missing; thecrowns of the 17, 15, 26, 27, 36, 37, 45,47 teeth are worn by 2/3 of their height;the chewing surfaces of these teeth aresmooth and pigmented; alveolar boneis not hypertrophied, interalveolar hei-ght is decreased. Specify the form ofpathological dental abrasion in thispatient:

A. Horizontal, uncompensated, IIIgrade of severityB. Horizontal, compensated, III gradeof severityC. Vertical, uncompensated, III gradeof severityD. Vertical, compensated, III grade ofseverityE. Combined, uncompensated, IIIgrade of severity

77. Following the extraction of the 12,11, 21, 22, 23 teeth, a 50-year-old pati-

Page 13: Krok 2 - 2014 Question Paper (Stomatology)

Кrok 2 Stomatology (англомовнi студенти) 2014 рiк 13

ent will receive an immediate-insertiondenture. What denture should be usedin this situation?

A. Laminar dentureB. Clasp dentureC. Fixed bridge dentureD. Adhesive dentureE. Removable bridge denture

78. A 10-year-old child was undergoi-ng complex dental care. During theprocedures, a deep carious cavitycommunicating with the tooth cavi-ty was revealed in the 36 tooth. Thetooth was not sensitive either to probi-ng, percussion, or temperature stimuli.The patient was diagnosed with chronicperiodontitis. During the treatment,the accidental perforation of the toothcavity floor occurred. What measuresshould be taken in this case?

A. Sealing the perforation with glassionomerB. Potassium iodide electrophoresisC. Application of zinc-eugenol pasteD. Sealing the perforation with calciumhydroxideE. Tooth extraction

79. Dental examination of a 19-year-oldgirl revealed white spots in the cervi-cal zone of the 11, 21 teeth. Accordi-ng to the patient, the spots appearedabout two months ago and had slowlyenlarged since that time. Which of thefollowing diseases is characterized bysuch history?

A. CariesB. Enamel hypoplasiaC. NecrosisD. FluorosisE. Erosion of hard tissues

80. A 52-year-old patient complai-ns of pain and a swelling in the ri-ght parotid region. These manifestati-ons have been present for about 2years. Over the last month the swelli-ng has grown bigger, pain has intensifi-ed. Objectively: the face is asymmetricdue to a dense infiltrate in the rightparotid region. The poorly circumscri-bed, tender to palpation lump infi-ltrates the surrounding tissues. On theright side of neck in front and behindthe sternocleidomastoid muscle there

are enlarged, dense, mobile lymphnodes. The right naso-buccal grooveis flattened, the corner of mouth isdownturned. The mouth opens freely.The are pronounced symptoms of theright facial nerve paresis. What diseasecan be suspected?

A. Adenocarcinoma of the parotidsalivary glandB. Chronic parotitisC. Actinomycosis of the parotid-masseteric regionD. Chronic lymphadenitisE. Pleomorphic adenoma of the parotidgland

81. A 24-year-old patient was undergoi-ng the extraction of the 25 tooth rootsfor the exacerbation of chronic peri-odontitis. During the operation theroots were dislocated into the maxillarysinus. What would be the surgeon’soptimal tactics?

A. To perform maxillary sinusotomyand remove the roots in hospital envi-ronmentB. Not to remove the roots, suture theextraction socketC. To remove the roots through theextraction socketD. To pack the extraction socketE. To perform maxillary sinusotomyand remove the roots in the outpatientdepartment

82. A 55-year-old male patient presentsto a dental clinic complaining of inabili-ty to use complete removable denturesfabricated a week ago. The patient alsopresents with muscle pain in the regionof the temporomandibular joint, toothchatter during a conversation, and di-fficult biting on food. Objectively: theface is elongated, nasolabial and labi-omental folds are flattened, the chin isretruded, the lips do not close. What isthe cause of these disorders?

A. Increased occlusal vertical dimensi-onB. Descending occlusionC. Inadequate fixation of denturesD. Left displacement of the mandibleE. Right displacement of the mandible

83. Parents of a 10-year-old boyconsulted an orthodontist about misali-

Page 14: Krok 2 - 2014 Question Paper (Stomatology)

Кrok 2 Stomatology (англомовнi студенти) 2014 рiк 14

gnment of the 21 tooth. Objectively:the 21 tooth is in a vestibular positi-on, there is enough space for it in thedental arch. What additional methodsof examination should be applied tospecify the treatment plan?

A. X-rayB. Pont’s index measuringC. Korkhaus’ index measuringD. ParallelingE. Masticaciography

84. A 38-year-old chemical plantworker, a smoker, with a history ofchronic enterocolitis, was undergoingpreventive medical examination. In theposterior third, along the midline of thetongue dorsum there is an area with nopapillae. The area is 2x1cm large, sli-ghtly dense on palpation. What is themost likely diagnosis?

A. Rhomboid glossitisB. Desquamative glossitisC. Chronic atrophic candidiasisD. Secondary syphilisE. Moeller-Hunter glossitis

85. A female patient presented to adental prosthetics clinic complaining ofacute pain in the 26 tooth. The paingets worse in the evening and at night.Objectively: the patient has a cantileverporcelain-fused-to-metal dental bridgesupported by the 26 tooth and the25 tooth cantilever. The tooth is live,the preparation was performed underconduction anesthesia. The prosthesisis fabricated in compliance with allrequirements. What medical error mi-ght have caused this complication?

A. Continuous tooth preparation wi-thout cooling, no provisional crownsB. Shaping the tooth as a truncatedconeC. Preparation of the radial shoulder inthe cervical zoneD. Lack of medial support for thebridgeE. Cementing the bridge on thephosphate cement

86. A 64-year-old patient had amyocardial infarction a month ago.He presents to a dental office forthe treatment of pulpitis of the 12tooth. Select the optimal method of

anesthesia:

A. Premedication + anesthetic withouta vasoconstrictorB. An anesthetic with a vasoconstrictorC. Premedication + an anesthetic wi-thout a vasoconstrictorD. Premedication + an anesthetic with avasoconstrictorE. Premedication + anesthetic with avasoconstrictor

87. A 45-year-old female patientcomplains of fever, appearance of rashin the mouth cavity and on the lips.She has a 4-year history of this di-sease which occurs mainly in autumnand spring. Objectively: oral mucosais hyperemic, there are painful erosi-ons covered with gray plaque. Thereare also hemorrhagic crusts on the lips.The skin of the forearm flexor surfacesexhibits maculo-papular rash. What isthe most likely diagnosis?

A. Erythema multiformeB. Dermatitis herpetiformisC. Chronic recurrent herpesD. Bullous pemphigoidE. Pemphigus vulgaris

88. A 27-year-old female patientconsulted a dentist about pain in the35 tooth. The pain is caused by coldstimuli and quickly abates after the sti-mulus is removed. Objectively: in thecervical region of the 35 tooth thereis a dental tissue defect within theenamel. The defect has white crumbli-ng edges. Electroexcitability of thepulp was 5 mkA. What is the most li-kely diagnosis?

A. Acute superficial cariesB. Local enamel hypoplasiaC. Acute initial cariesD. Enamel necrosisE. Enamel erosion

89. A patient has an indication for the12 tooth extraction for the exacerbati-on of chronic periodontitis. It was deci-ded to apply infraorbital and incisi-ve anesthesia in this patient. Whatnerve structures are blocked by thisanesthesia?

Page 15: Krok 2 - 2014 Question Paper (Stomatology)

Кrok 2 Stomatology (англомовнi студенти) 2014 рiк 15

A. Anterior superior alveolar branches,n. IncisivusB. Middle superior alveolar branches, n.NasopalatinusC. Posterior superior alveolar branches,n. IncisivusD. Greater palatine nerve, middlesuperior alveolar branchesE. Nasopalatine nerve, anterior superi-or alveolar branches

90. A 52-year-old patient consulted adentist about a tumor-like neoplasm inthe region of the lower lip vermilionborder. Examination of the vermilionborder on the left revealed a roundishtumor about 0,5 cm in diameter on athin pedicle. The tumor had a smoothsurface, was mobile, soft, and painless.What is the most likely diagnosis?

A. Papilloma of the lower lipB. Cutaneous hornC. LeukoplakiaD. Verrucous precancerous lesion of thevermilion borderE. Abrasive Manganotti precancerouscheilitis

91. A 21-year-old male patientundergoes treatment for chroniccatarrhal gingivitis. There is a signifi-cant amount of soft dental plaque, hi-gh speed of its production. The patientknows how to practice good oral hygi-ene. Recommend a mouthrinse for thispatient:

A. ChlorhexidineB. Dimethyl sulfoxideC. Mefenaminum natriumD. Hydrogen peroxideE. Furacilin

92. A male patient should have theroot of the 17 tooth removed prior toprosthetic procedures. The crown ofthe tooth is decayed beneath the gumline. What tool should be used?

A. Bayonet forcepsB. S-shaped forcepsC. Straight forceps with convergingbeaksD. S-shaped right forcepsE. Straight forceps with non-convergingbeaks

93. A patient consulted a dentist about

pain in the 47 tooth. X-ray examinati-on revealed fragments of endodonticinstruments in the canals of the medialand distal root. In the periapical zonethere are well-defined areas of bonedestruction. Select the most appropri-ate method of treatment:

A. Tooth extractionB. RadectomyC. Tooth replantationD. Tooth hemisectionE. Conservative treatment

94. A 50-year-old male patient witha defect of the hard palate needs apalatal plate. Objectively: the maxillaryteeth are intact. What method of fixati-on should be used?

A. Mesio-distal claspsB. Supporting claspsC. AttachmentD. Bar fixationE. Telescopic system

95. A 40-year-old male patient presentswith vicious union after a fracture ofthe maxilla. Objectively: the remaining13 and 14 teeth are displaced in a distaldirection. What method of orthopedictreatment would be most adequate inthis case?

A. Removable double-arch dentureB. Cast clasp dentureC. Removable laminar dentureD. Removable denture with metal baseE. Removable denture with bilayer base

96. Following the unilateral resection ofthe maxilla a 52-year-old patient recei-ved the immediate-insertion denture.What is the term of its use?

A. 1-3 monthsB. 1-2 monthsC. 7-12 monthsD. 2-3 yearsE. 4-5 years

97. A 28-year-old patient complainsof pain in the 37 tooth when eatinghot food. Objectively: the 37 tooth haschanged in color, there is a carious cavi-ty filled with softened dentin, the toothcavity is open, probing of the root pulpcauses pain response, percussion causesno pain. What is the most likely di-

Page 16: Krok 2 - 2014 Question Paper (Stomatology)

Кrok 2 Stomatology (англомовнi студенти) 2014 рiк 16

agnosis?

A. Chronic gangrenous pulpitisB. Exacerbation of chronic periodontitisC. Exacerbation of chronic pulpitisD. Chronic fibrous pulpitisE. Chronic fibrous periodontitis

98. A 16-year-old female patientcomplains of gingival pain andhemorrhage, a gingival deformity. Overthe last 2 years the gums periodicallybleed during tooth brushing. Objecti-vely: in the region of the front mandi-bular teeth the gingival papillae areenlarged and reach 1/2 of the toothcrown height. The gums are swollenand tender, bleed when touched. Radi-ographic alterations were not revealed.What is the most likely diagnosis?

A. Hypertrophic gingivitisB. Ulcerative gingivitisC. Generalized periodontitisD. Catarrhal gingivitisE. Epulis

99. A patient presented to a dental cli-nic for complex dental care. Objecti-vely: the 37 tooth exhibits a deep cari-ous cavity communicating with thetooth cavity. There is no response tostimuli. Radiography reveals wideni-ng and deformation of the periodontalligament space in the apical region.What is the most likely diagnosis?

A. Chronic fibrous periodontitisB. Chronic granulating periodontitisC. Chronic granulomatous periodontitisD. Chronic fibrous pulpitisE. Chronic gangrenous pulpitis

100. A 49-year-old patient consulted adental surgeon about complex dentalcare. He has an indication for theextraction of the 16 tooth. History: thetooth hasn’t been treated before, it hasdecayed within the last 4 years. Objecti-vely: the 16 tooth crown is decayedby over 2/3, the mucosa exhibits nopathological changes. What tool isrequired for the tooth extraction?

A. Bayonet root forcepsB. S-shaped forceps (right)C. Straight elevatorD. S-shaped closed-beak forcepsE. Crown bayonet forceps

101. A 16-year-old female patientcomplains of a deformation andrestricted mouth opening since birth.Objectively: the face is symmetrical, di-sproportionate, there is a bird-like facesymptom. The mouth opens up to 1 cm.The patient has an occlusal abnormali-ty, namely deep incisal overbite. Whatpathology is found in this patient?

A. Micrognathia with congenitalankylosisB. Micrognathia with acquired ankylosisC. Maxillary protractionD. PrognathismE. Mandibular protraction

102. A 13-year-old boy complains ofpain in the region of the extracted 46tooth, irradiating to the ear and temple;halitosis. The tooth was extracted3 days ago. Objectively: submandi-bular lymph nodes are enlarged,tender on palpation. Mucosa aroundthe extracted tooth is hyperemic,edematous. The socket of the 46 toothis filled with a gray clot. What is themost likely diagnosis?

A. Alveolitis of the extracted 46 toothB. Acute odontogenous mandibularosteomyelitis starting from the 46 toothC. Acute odontogenous lymphadenitisof the right submandibular regionstarting from the 46 toothD. Acute odontogenous mandibularperiostitis starting from the 46 toothE. Neuralgia of the III branch of thetrigeminal nerve

103. A 47-year-old male patientconsulted a dental surgeon about aslowly growing tumor on his lowerlip which appeared six months ago.Objectively: on the lower lip there isa single horny conical projection abovethe surface of the skin up to 1,5 cmwide. The neoplasm adheres to theskin. Palpation reveals dense kerati-nous masses. What is the provisionaldiagnosis?

Page 17: Krok 2 - 2014 Question Paper (Stomatology)

Кrok 2 Stomatology (англомовнi студенти) 2014 рiк 17

A. Cutaneous hornB. Verrucous precancerous lesion of lipC. Circumscribed hyperkeratosisD. Papilloma of lipE. Abrasive precancerous Manganotticheilitis

104. A 45-year-old patient has a densesemicircle neoplasm 1 cm in diameteron his lower lip. The tumor projectsabove the vermilion border by 0,5 cm.The surface is grayish- pink with asmall number of closely arranged whi-tish scales. The tumor is not tender onpalpation. What is the most likely di-agnosis?

A. Verrucous precancerous lesion of thevermilion borderB. Verucca vulgarisC. PapillomaD. Pyogenic granulomaE. Manganotti cheilitis

105. Parents of an 8-year-old childcomplain about the presence of chalkyspots on the child’s front maxillaryteeth which have recenly erupted.Objectively: the vestibular surfaces ofthe 11 and 12 teeth exhibit white spotsin the cervical zone. Enamel at these si-tes is not transparent. What additionaltest can be used?

A. Staining with methylene blueB. RadiographyC. Staining with iodine-containingsolutionsD. Electric pulp testE. Radiovisiography

106. An 11-year-old boy complains ofhaving a painful lesion in the mouthcavity, which makes it difficult to eat.Such symptoms were first noticed twoyears ago. Examination of the mouthfloor reveals a tender oval erosion si-zed up to 0,8 cm, and covered withgrayish-white plaque. The erosion hasa hyperemic rim. Make a provisionaldiagnosis:

A. Chronic recurrent aphthous stomati-tisB. Erythema multiformeC. Behcet’s syndromeD. Stevens-Johnson syndromeE. Traumatic erosion

107. A 3-year-old girl has been di-agnosed with acute odontogenicmandibular periostitis beginning fromthe 74 tooth. It is required to performperiostomy and extract the 74 tooth.The child is excited. Select the optimaltype of anesthesia for the surgery:

A. PhlebonarcosisB. Inferior alveolar nerve blockC. Mask anesthesiaD. Intubation anesthesiaE. Central anesthesia

108. A 7-year-old child has been di-agnosed with chronic periodontitis ofthe 64 and 65 teeth. The child has ahistory of hemophilia A. What is themedical tactics of choice?

A. Extraction of the affected teeth inthe hematology department after takingthe appropriate preparatory measuresB. Extraction of the affected teethis possible both on an in-patient andout-patient basisC. Extraction of the affected teeth on anout-patient basis with further packingof alveolar sockets with a hemostaticspongeD. Extraction of the affected teeth inthe maxillofacial departmentE. Extraction of teeth is contraindicatedbecause of the high probability ofbleeding

109. A 3-year-old child has a hardand soft palate defect. It is plannedto fabricate a "floating"Cese obturator(Chasovskaya modification). Whatimpression material should be used?

A. Thermoplastic or siliconeB. StomalginC. GypsumD. HydrocolloidE. Eugenol-zinc

110. Examination of a child hospitali-zed for scarlet fever revealed alterati-ons of the tongue mucosa which turnedbright red, glossy, and dry. At thesame time the fungiform papillae wereenlarged. The described appearance oftongue is referred to as:

Page 18: Krok 2 - 2014 Question Paper (Stomatology)

Кrok 2 Stomatology (англомовнi студенти) 2014 рiк 18

A. StrawberryB. PlicatedC. RhomboidD. GeographicE. Black

111. A 64-year-old patient withedentulous jaws has a mild uni-form atrophy of the maxilla anda pronounced atrophy of lateralsegments of the mandible. The mucosain these regions makes longitudinalfolds, the alveolar crest is mobile in theanterior part. The condition of mandi-ble should be taken into considerationat the following stage of fabrication ofa complete removable denture:

A. Taking a differentiated functionalimpressionB. Taking an anatomical impressionC. Taking a compression functionalimpressionD. Taking a decompression functionalimpressionE. Determining the centric jaw relation

112. A 30-year-old female patientcomplains of limited mouth opening.She has a history of an injury receivedat the time when her jaws were closed.Objectively: mouth opening is limitedto 1,5 cm, the bite is normal, mouthopening causes the displacement ofmandible to the left. Palpation revealspain in the left temporomandibular joi-nt (TMJ). X-ray picture confirms boneintactness. What is the most likely di-agnosis?

A. Acute traumatic arthritis of the leftTMJB. Fracture of the left articular processof the mandibleC. Acute unilateral dislocation of theTMJD. Contusion of the left TMJE. Fracture of the left zygomatic bonewith a displacement of bone fragments

113. A 41-year-old patient has been di-agnosed with a fracture of the mandi-bular alveolar process. Objectively:the teeth are intact. When the jawsare closed, the multipoint occlusalcontact is observed. The patient is tobe transported to a specialized hospi-tal. What kind of first aid is indicated in

this case?

A. Stiff head-chin strap by D.O. EntinB. Smooth braceC. Standard Vasilyev splintD. Tigerstedt’s splint with wire loopsE. Ivy ligature

114. In a 7-year-old child the rightmandibular molars overlap the maxi-llary molars, there are no other occlusi-on abnormalities. Size and shape of thelower dental arch are normal. Specifythe appliance for the treatment of thisabnormality:

A. Upper-jaw appliance with a sectorexpansion screwB. Upper-jaw appliance with a middleexpansion screwC. Upper-jaw appliance with a rightguide planeD. Upper-jaw appliance with a leftguide planeE. Angle’s coil spring

115. Parents of a 5-year-old childconsulted an orthodontist about mi-spronunciation of sounds by the chi-ld. Objectively: the child’s face isunremarkable. The patient has deci-duous dentition. There are 1-1,5 mmgaps in the frontal segment from 53to 63. Occlusion in the sagittal andtransversal planes is normal. Whattype of Frankel’s functional regulatoris used to treat the above-describedmalocclusion?

A. Type IVB. Type IIIC. Type IID. Type IE. -

116. A 35-year-old female patient seeksprosthetic dentistry. Objectively: the18, 14, 13, 12, 11, 21, 22, 23, 24, 28 teethneed restoration. The crowns confini-ng the defect are high and stable. Torestore the integrity of the dental arch,it is planned to fabricate a cast claspdenture. What kind of mechanical fi-xator should be used?

Page 19: Krok 2 - 2014 Question Paper (Stomatology)

Кrok 2 Stomatology (англомовнi студенти) 2014 рiк 19

A. Bar fixationB. Supporting-retaining claspsC. Retaining claspsD. Telescopic fixationE. Dentoalveolar clasps

117. A 48-year-old patient hasundergone unilateral resection ofthe maxilla. He needs a resectionprosthesis. Objectively: the 21, 22, 23,24, 25, 26 teeth are stable, hard palateis broad, flat. What kind of attachmentof clasps to the base will reduce theinstability of the resection prosthesis?

A. ArticularB. StableC. ElasticD. FixedE. Does not matter

118. Residents of a remote settlementneed prosthetic care. What health carefacility should provide it?

A. Regional dental clinicB. City dental clinicC. Occupational health facilityD. Rural district hospitalE. Rural outpatient clinic

119. A 24-year-old hemophiliac hasbeen diagnosed with exacerbation ofchronic pulpitis of the 11 tooth. Selectthe best method of tooth treatment inthis case:

A. Devital extirpationB. Biological treatmentC. Vital amputationD. Devital amputationE. Vital extirpation

120. A 25-year-old female patientconsulted a dentist about acute painin the maxilla on the left. The painoccurs while eating. Objectively: on thedistal approximate surface of the 26tooth there is a cavity filled with lightsoftened dentin. Probing causes a slightpain along the dentin-enamel juncti-on, percussion causes no pain response.Cold water causes quickly abating pain.What is the most likely diagnosis?

A. Acute median cariesB. Chronic median cariesC. Acute deep cariesD. Chronic fibrous pulpitisE. Chronic deep caries

121. A 45-year-old male patientcomplains of bleeding gums, toothmobility, hypersensitivity in the cervicalzone. Objective examination revealedpresence of all teeth, 1-2 class mobi-lity, pathological gingival pockets wi-th serous exudate. Radiograph showswidening of the periodontal ligamentspace, alveolar bone resorption by 1/3- 1/2 of the root length. What kind oftooth stabilization is indicated?

A. Stabilization of entire dental archB. FrontC. Front and sagittalD. SagittalE. Parasagittal

122. During mandibular anesthetizati-on with 4% solution of Ultracain-forte, a 13-year-old girl complained ofsudden weakness, laboured respiration,nausea, fear. Objectively: the patienthas pale skin covered with cold sweat,hyperemic spots on the face and neck.Respiration is frequent, the pulse isweak, blood pressure is 60/40 mm Hg,hands are cold. What is the most likelydiagnosis?

A. Anaphylactic shockB. Toxic shockC. SyncopeD. Traumatic shockE. Collapse

123. A 65-year-old female patientcomplains of complete edentulousness.Examination of the oral cavity revealedthat alveolar process of the edentulousmandible was markedly atrophiedin the frontal region, while it wasexpressed in the distal region. Specifythe class of atrophy according to Kellerclassification:

A. Class IVB. Class IIC. Class ID. Class IIIE. Class I and III

124. A 28-year-old patient complains

Page 20: Krok 2 - 2014 Question Paper (Stomatology)

Кrok 2 Stomatology (англомовнi студенти) 2014 рiк 20

of tender gums, gingival hemorrhagesand pain, especially while eating,for a week. Objectively: the gumsare swollen and markedly hyperemic,especially in the region of the 43, 42, 41,31, 32, 33 teeth, soft plaque and tartarare present, gingival sulci are up to 2mm deep. What is the most likely di-agnosis?

A. Acute deep gingivitisB. Chronic catarrhal gingivitisC. Hypertrophic gingivitisD. Initial stage of chronic generalizedperiodontitisE. Exacerbation of initial generalizedperiodontitis

125. A 24-year-old male patientcomplains of acute gingival pain, theinability to eat, bad breath, fever upto 38oC, general weakness. Objecti-vely: the patient is pale, lymph nodesare enlarged. Gingival papillae areswollen and hyperemic, there are indi-vidual ulcers covered by necrotic coati-ng. What additional study is requiredin the first place in order to clarify thediagnosis?

A. Complete blood countB. UrinalysisC. Panoramic radiographyD. Allergologic testsE. Kavetsky test with trypan blue

126. A 35-year-old patient complai-ns of gingival hemorrhage duringtooth brushing. Objectively: gums ofboth jaws are hyperemic and cyanotic,supragingival and subgingival tartar ispresent. The periodontal pockets are4-6 mm deep. What is the most likelyprovisional diagnosis?

A. II grade generalized periodontitis,chronic courseB. Catarrhal gingivitisC. Hypertrophic gingivitisD. Exacerbation of I grade generalizedperiodontitisE. II grade parodontosis

127. A 27-year-old male patientcomplains of aching long-lasting painin the 15 tooth during having meals,especially cold food. Sometimes thepain occurs when the temperaturechanges. Objectively: on the distal

surface of the 15 tooth there is a cavi-ty filled with softened dentin. Probingcauses pain reaction. Electroexcitabili-ty of the pulp is 35 µA. What is the mostlikely diagnosis?

A. Chronic fibrous pulpitisB. Acute deep cariesC. Chronic deep cariesD. Hyperemia of the pulpE. Exacerbation of chronic pulpitis

128. A 25-year-old patient undergoestreatment at the clinic of maxi-llofacial surgery for the mandibularameloblastoma localized in the angle.The patient has been scheduled for apartial resection of the mandible with asingle-stage bone autoplasty. It is mostexpedient to take a bone graft from:

A. Iliac bone ridgeB. CollarboneC. Foot bonesD. FemurE. VI rib

129. A 36-year-old female patientcomplains of headache, inability toclose the jaws. She has a history ofan injury followed by loss of consci-ousness, nausea. Objectively: the mi-dface tissues are edematous, there isbilateral eyelid hemorrage, open bite.Palpation of the midface bones causesacute pain. In the back of nose thereis marked symptom of step deformity,nasal hemorrhage is present. Palpationof the oral cavity reveals mobility of themaxilla. Make a diagnosis:

A. Le Fort II maxillary fracture (mi-ddle), concussionB. Le Fort III maxillary fracture (top)C. Le Fort I maxillary fracture (bottom)D. Le Fort II maxillary fracture (mi-ddle)E. Maxillary Guerin fracture

130. Objective examination of a 65-year-old patient with the completelyedentulous mandible revealed amarked uniform atrophy of thealveolar bone; bony prominences onthe lingual surface in the regionwhere premolars had previously been.Mucosa was unevenly pliable; alveolar

Page 21: Krok 2 - 2014 Question Paper (Stomatology)

Кrok 2 Stomatology (англомовнi студенти) 2014 рiк 21

crest was mobile in the frontal region.The clinical condition of the mandibleshould be taken into consideration atthe following stage of denture fabri-cation:

A. Taking a differentiated impressionB. Taking an anatomical impressionC. Taking a positive pressure impressionD. Taking a decompression impressionE. Measuring the centric relation of jaws

131. Objective examination of a 67-year-old patient with the edentulousmaxilla revealed minor uniformatrophy of the alveolar processes.Maxillary prominences were wellpreserved, the frenulum and bucco-alveolar folds were attached at thebase of the alveolar process, the palatewas deep, the torus palatinus wasexpressed insignificantly. These clini-cal presentations correspond with thefollowing class of atrophy according toSchroeder classification:

A. 1B. 2C. 3D. 4E. 5

132. A 2-year-old boy has a brownspot with a rough surface covered withcoarse hair on his left cheek. Accordi-ng to parents, the neoplasm has beenpresent since birth and exhibits a slightenlargement as the child grows. Whatis the most likely diagnosis?

A. Pigmented nevusB. Vascular nevusC. Kaposi’s sarcomaD. Facial neurofibromatosisE. Melanoma of cheek

133. A 53-year-old male patient hasa history of generalized periodontitis.It is planned to fabricate fixed splintsfor both jaws intended to stabilize theteeth along the entire dental arch. Whi-ch of the maxillary pillars will have thefunctional significance for the distri-bution of masticating pressure?

A. Frontonasal, zygomatic,pterygopalatine, palatineB. Frontonasal, zygomatic,pterygopalatineC. Zygomatic, pterygoid, palatineD. Frontonasal, pterygoid, palatineE. Frontonasal, zygomatic, palatine

134. A 12-year-old girl complains ofwhite spots on the vestibular surfacesof teeth and mouth soreness thatoccurs during eating sour foods. It isknown from the history that the spotsappeared about 3 months ago. Objecti-vely: the vestibular surfaces of the frontmaxillary teeth exhibit chalky spotsthat are stainable with 2% methyleneblue. What is the most likely diagnosis?

A. Acute initial cariesB. Acute superficial cariesC. Acute median cariesD. Chronic median cariesE. Chronic superficial caries

135. A 25-year-old male patient hasbeen diagnosed with a linear nondi-splaced fracture of the mandiblebetween the 32 and 33 tooth. Whatsplint should be used for the fixationof bone fragments?

A. Smooth braceB. Splint with spacer barC. Brace with an oblique planeD. Vasilyev splintE. Wire anchor splint

136. A 46-year-old patient complai-ns of a periodic swelling in the rightsubmandibular region when eating, drymouth. Bimanual palpation of the mi-ddle portion of the hyoid region onthe right revealed a moderately pai-nful dense lump. From the excretoryduct of the submandibular gland somemuco-purulent exudate is expressed.The patient should be referred to thefollowing roentgenologic examination:

A. Radiography of the mouth floorB. Panoramic radiography of the jawsC. Radiography of the mandible in thelateral projectionD. Survey radiography of the facialbonesE. Spot-film radiography of the alveolarprocess

Page 22: Krok 2 - 2014 Question Paper (Stomatology)

Кrok 2 Stomatology (англомовнi студенти) 2014 рiк 22

137. An 8-year-old boy complains of adefect in the 11 tooth crown. Objecti-vely: 1/3 of the 11 tooth crown is brokenoff, the pulp chamber is closed. Radi-ograph shows incomplete root formati-on. Select the optimal way of prostheti-cs for the 11 tooth:

A. Thin-walled metal crownB. Metal-ceramic crownC. Plastic crownD. Post and coreE. Combined crown

138. A 9-year-old boy requires complexdental care. Objective examinationrevealed chalky spots with lustroussurface on the vestibular surfaces ofthe 22, 21, 11, 12 teeth, as well as onthe tubercles of the 26, 26, 46 teeth.The affected teeth are not sensitive tothermal and chemical stimuli. The chi-ld has a previous history of rickets. Helives in a locality where the fluorideconcentration in drinking water is 0,5mg/l. What is the most likely diagnosis?

A. System enamel hypoplasiaB. FluorosisC. Stainton-Capdepont syndromeD. Local enamel hypoplasiaE. Amelogenesis imperfecta

139. A 6-year-old child with contusedforehead laceration had been takento the emergency room at a chi-ldren’s hospital. The child was exami-ned by the maxillofacial surgeon whoperformed primary surgical debri-dement and closed the wound withimmediate primary sutures. The childwas recommended to undergo furthertreatment on an outpatient basis ata district polyclinic. Specify the first-priority measure in this case:

A. Consultation by a neurosurgeonB. Surgeon follow-upC. Complex dental careD. Antimicrobial therapyE. Bed rest

140. An 8-year-old child has been clini-cally diagnosed with exacerbation ofchronic periodontitis of the 84 tooth.The crown is decayed by half. What isthe dentist’s optimal tactics?

A. ExtractionB. Endodontic treatmentC. Endodontic treatment and drugtherapyD. Drug therapyE. Dissection along the mucogingivaljunction, drug thrapy

141. 6 months ago a 65-year-old pati-ent received a permanent intracoronalsplint for the front maxillary teeth. Thesplint was fabricated with the use offiberglass tape. Radiograph shows thatthe root of the 12 tooth is localized onlyin the soft tissues. The splint is in a goodcondition. What is the optimal tactics ofthis patient management?

A. Amputate and extract the root of the12 toothB. Extract the 12 tooth and fabricate abridgeC. Start conservative treatmentD. Replace the 12 tooth in the splint byan artificial plastic toothE. Extract the 12 tooth and restore thedefect with a porcelain-fused-to-metalcrown

142. A 55-year-old male patient di-agnosed with chronic median caries ofthe 33 tooth presented to a dental cli-nic. It is known from the history thatthe patient has a cardiac pacemakerWhat would be the optimal materialfor a permanent filling?

A. Chemical-cure compositeB. Silicate cementC. Silicophosphate cementD. Light-curing compositeE. Compomer

143. A 14-year-old patient complains ofmoderate non-irradiating pain in theregion of the 37 tooth. Objectively: the37 tooth exhibits a deep carious cavitycommunicating with the pulp chamber.Probing and thermal stimulation causeno pain response, vertical percussi-on is positive. Radiograph shows nopathological alterations. What is themost likely diagnosis?

Page 23: Krok 2 - 2014 Question Paper (Stomatology)

Кrok 2 Stomatology (англомовнi студенти) 2014 рiк 23

A. Acute serous periodontitisB. Acute serous periostitisC. Acute suppurative pulpitisD. Acute suppurative periodontitisE. Exacerbation of chronic periodontitis

144. A 32-year-old patient lost the 41and 31 teeth as a result of a sportsinjury. Reimplantation is impossiblebecause the crown and root surfacesof these teeth are cracked and chipped.Clinical and radiological examinationproved the possibility of immediateimplantation. What material should bepreferred for the implants?

A. TitaniumB. CarbonC. PorcelainD. PlatinumE. Sapphire

145. Radiography of the 46 tooth in a7-year-old child revealed parallel rootwalls with a gradual decrease of theirthickness resulting in pointed ends.There is a root canal narrowing in theregion that is adjacent to the tooth cavi-ty, and a widening at the nascent apicalaperture. Periodontal ligament spacehas a uniform width over the entireroot length. In the periapical region itmerges with the growth zone. Specifythe stage of the root development:

A. Open apexB. Incomplete root growthC. Complete root and periodontdevelopmentD. Incomplete periodont developmentE. -

146. A 5-year-old child complains ofspontaneous pain in a maxillary toothon the right. The pain gets worse atnight and during eating cold food.Objectively: the 65 tooth has a deepcarious cavity communicating withthe tooth cavity. Probing causes painresponse, the tooth is not sensitive topercussion. Cold water causes long-lasting pain. Make a provisional di-agnosis:

A. Exacerbation of chronic pulpitisB. Acute periodontitisC. Exacerbation of chronic periodontitisD. Acute serous pulpitisE. Acute purulent pulpitis

147. A 4-year-old child complains ofacute spontaneous pain in a mandi-bular tooth on the right. The pain isaggravated by cold stimuli. Objecti-vely: the 84 tooth exhibits a deep cari-ous cavity that does not communicatewith the tooth cavity. Probing causespain response all over the cavity floor,percussion causes pain. The mucosa inthe region of the 84 tooth is hyperemic,edematous. Regional lymph nodes areenlarged. What is the most likely di-agnosis?

A. Acute pulpitis complicated by peri-odontitisB. Acute serous pulpitisC. Acute serous periodontitisD. Exacerbation of chronic periodonti-tisE. Acute suppurative pulpitis

148. A patient presented to a dentalclinic for the purpose of prostheti-cs. Objectively: completely edentulousmandible. There is marked and uni-form atrophy of the alveolar portion.Frenula and folds are characterizedby high attachment. Specify the typeof mandibular atrophy according toKeller classification:

A. Class IIB. Class IC. Class IIID. Class IVE. Class V

149. A 43-year-old patient consulteda dentist about numbness of the lefthalf of his tongue, which developed 10days ago after the atypical extraction ofthe 38 tooth. What nerve was damagedduring the surgery?

A. LingualB. SublingualC. Inferior alveolarD. FacialE. Glossopharyngeal

150. Soon after anesthetization a55-year-old patient reported severe

Page 24: Krok 2 - 2014 Question Paper (Stomatology)

Кrok 2 Stomatology (англомовнi студенти) 2014 рiк 24

weakness, retrosternal pain that irradi-ated to his left arm and scapular region,palpitations. Objectively: the patient isconscious, adynamic, pale, the foreheadis covered with cold sweat. AP- 90/60mm Hg, cardiac sounds are muffled, thepulse is thready and arrhythmic. Whatcondition developed in the patient?

A. Myocardial infarctionB. Cardiogenic form of anaphylacticshockC. Angina attackD. CollapseE. -

151. During tooth extraction a 55-year-old female patient reported dullretrosternal pain, feeling of chestcompression. The dental surgeon whowas performing the tooth extractionmade a diagnosis of angina. What drugsshould be given the patient for the reli-ef of this condition?

A. Nitroglycerin, validolB. AnalginC. BaralginD. DiphenhydramineE. Ketanov

152. A 1-month-old child has beenbrought to a maxillofacial surgeon forexamination. Objectively: there is a fi-ssure running transversely from the ri-ght corner of mouth through the fullthickness of cheek up to the anterioredge of the masseter muscle. What isthe patient’s diagnosis?

A. Right transverse facial cleftB. Right oblique facial cleftC. Cleft nasal deformityD. Right upper lip cleftE. Mandibular cleft

153. A 38-year-old female complai-ns of experiencing pain in her lowerlip for half a year, difficult eating andtalking. The patient sought medicalhelp at place of residence, but theadministered treatment was ineffective.Examination of the lower lip revealeda deep fissure in the centre whichaffected the vermilion border andpartly the mucous membrane of lips.The fissure was covered with a browncrust, after its removal the fissurestarted bleeding. The fissure edges are

hyperkeratinized, infiltrated. Select theoptimal treatment tactics:

A. Fissure excisionB. Application of keratoplastic agentsC. Irradiation with helium-neon laserD. Application of corticosteroid oi-ntmentE. Application of antibiotic ointment

154. A 48-year-old female patientcomplains of hard tissue defects foundon the maxillary incisors. The defectsappeared about a year ago and havebeen enlarging since that time. Objecti-vely: on the convex portion of the vesti-bular surface of the maxillary incisorsthere are oval varisized enamel defects.The defect surface is smooth, lustrous,solid. In order to prevent progressionof this process, the patient should beadvised to limit consumption of:

A. Sour foodB. Marine productsC. Meat productsD. Sweet foodE. Tea

155. During local anesthetization with2 ml of 10% lidocaine a 9-year-old girlsuddenly screamed, lost consciousnessand developed generalized convulsi-ons. The skin is pale and cyanotic.The pulse is not palpable because ofconvulsions. What is the provisional di-agnosis?

A. Lidocaine intoxicationB. AngioedemaC. Anaphylactic shockD. Bronchial obstructionE. Febrile seizures

156. A 56-year-old patient wasundergoing treatment for pulpitis ofthe 47 tooth. For the purpose ofthe pulp devitalization arsenic pastewas used. The patient failed to keepan appointment with the dentist forfurther treatment. As a result, toxicarsenic periodontitis developed. Whichof the physiotherapy methods shouldbe applied to address the complicati-on?

Page 25: Krok 2 - 2014 Question Paper (Stomatology)

Кrok 2 Stomatology (англомовнi студенти) 2014 рiк 25

A. Potassium iodide electrophoresisB. UHFC. D‘arsonvalisationD. FluctuorizationE. Galvanization

157. Following an uncomplicatedextraction of the 37 tooth, a 60-year-oldpatient has profuse phatnorrhagia. Thepatient has a 6-year history of essenti-al hypertension. AP- 180/110 mm Hg.What kind of emergency care shouldbe rendered?

A. Give an injection of antihypertensivedrugs and pack the tooth socket tightlyB. Pack the tooth socket with ahemostatic spongeC. Pack the tooth socket tightly with aniodoform turundaD. Suture the tooth socketE. Give an injection of hemostatic drugs

158. A child is 11 years old. Exami-nation of the oral cavity revealed thatthe front maxillary teeth completelyoverlap the mandibular ones. Lateralteeth in the sagittal and transversalplanes have a normal contact. The chi-ld has been diagnosed with a deepoverbite. This abnormality is typicallyaccompanied by the dysfunction of:

A. Chewing, biting offB. Breathing, swallowingC. Swallowing, biting offD. Biting off, breathingE. Speech, chewing

159. A 25-year-old male patientpresented to the department of maxi-llofacial surgery with complaints ofpain during eating, which arose threedays ago after an injury. Objectively:the face is asymmetric due to theswelling of the buccal and infraorbi-tal regions on the left. Mouth openingis slightly restricted because of pain.Maxillary alveolar process and leftmaxillary teeth are mobile. There is ahemorrhage along the left mucogingi-val junction in the region of the saidteeth; the site is tender to palpation.What diagnosis can be assumed?

A. Fracture of the maxillary alveolarprocessB. Le Fort II fracture of the maxillaC. Fracture of the zygomatic boneD. Fracture of the zygomatic archE. Le Fort III fracture of the maxilla

160. A 34-year-old patient complains ofintense, tearing, throbbing pain in the17 tooth, which appeared for the firsttime and persists for 4 days. Percussioncauses acute pain in any direction, thetooth is mobile, the mucosa around the17 tooth is hyperemic and edematous;palpation of the mucogingival junctioncauses acute pain. What is the most li-kely diagnosis?

A. Acute suppurative periodontitisB. Acute serous periodontitisC. Exacerbation of chronic periodontitisD. Localized periodontitisE. Acute suppurative pulpitis

161. A 45-year-old patient complains ofacute, spontaneous, nocturnal pain in amaxillary tooth on the right. The painarose three days ago, has a pulsatingnature, irradiates to the zygomatic regi-on and virtually doesn’t cease. Objecti-vely: there is a deep carious cavity inthe 15 tooth. Dentin is of dirty-graycolor, softened. Probing causes painresponse across the entire floor, thetooth is sensitive to percussion. Coldstimulus reduces the pain intensity.Electroexcitability of the pulp is 55 µA.What is the most likely diagnosis?

A. Acute suppurative pulpitisB. Acute pulpitis limitedC. Acute diffuse pulpitisD. Acute suppurative periodontitisE. Acute serous periodontitis

162. A 13-year-old girl complains ofexperiencing pain, gum bleeding andbad breath for about six months.Objectively: in the region of the frontmandibular teeth the gingival papillaeand gingival margin are hypertrophi-ed, edematous, cyanotic. There is a softplaque in the cervical region. Hygieneindex -3. The doctor administered alocal anti-inflammatory therapy. Speci-fy the further tactics of local treatment:

Page 26: Krok 2 - 2014 Question Paper (Stomatology)

Кrok 2 Stomatology (англомовнi студенти) 2014 рiк 26

A. SclerotherapyB. Gingival curettageC. GingivotomyD. PhysiotherapyE. Vestibuloplasty

163. A 6-year-old child complains ofpain in the right submandibular regi-on. A day before he complained of asore throat. Objectively: the child is inmoderately severe condition, body itis 37, 9oC. The face is asymmetric dueto a dense tender infiltrate in the rightsubmandibular region. The infiltrate isslightly mobile, the overlying skin ishyperemic. There is fluctuation in thecenter. The teeth are intact. What is themost likely diagnosis?

A. Acute suppurative non-odontogenicsubmandibular lymphadenitisB. Acute serous non-odontogenicsubmandibular lymphadenitisC. Phlegmonous adenitisD. Acute suppurative odontogenicsubmandibular lymphadenitisE. Sialadenitis

164. A 45-year-old male patient worki-ng as an actor consulted a dentistabout mobility of his front mandi-bular teeth, gingival hemorrhages duri-ng tooth brushing. Objectively: gingi-val hyperemia and hemorrhage, peri-odontal pockets in the front porti-on of the mandible are up to 5 mmdeep, tooth mobility of the I-II classis present. What kind of splinting themandibular front teeth will be optimalfor this patient?

A. Fiber-reinforced adhesive splintB. Cap splintC. Ligature wireD. Full crown splintE. Cast pin splint

165. A 44-year-old female patient wi-th generalized I grade periodontitis isscheduled for the selective grinding ofteeth. What examination is requiredprior to this procedure?

A. OcclusiographyB. GnathodynamometryC. RadiographyD. RheoparodontographyE. Electroodontometry

166. A 60-year-old patient undergoestreatment of the 11 and 21 tooth forchronic deep caries (Black’s class III).He has a history of cataract surgery.What is the optimal material for cavityfilling in this case?

A. Chemical cure compositeB. Hybrid glass ionomerC. CompomerD. Light cure compositeE. Silicate cement

167. Clinical examination of a 10-year-old girl’s oral cavity revealed an 11mm gap in a sagittal plane, the contactof the lateral maxillary teeth with thefront mandibular teeth, mesiobuccalcusps of the 16 and 26 tooth locatedon the cusps of the 35 and 45 tooth.What additional examination methodwill allow to make a definitive diagnosisand specify clinical form of malocclusi-on?

A. Profile teleroentgenographyB. OrthopantomographyC. Anthropometric measuring of jawmodelsD. Determining morphological facialindexE. Clinical functional tests

168. A 70-year-old completelyedentulous patient undergoes treatmentwith complete removable dentures.Artificial teeth are placed upon aspherical surface. Specify the averageradius of the spherical surface thatwould ensure close teeth contact duri-ng the mandible motions:

A. 9 cmB. 5 cmC. 7 cmD. 12 cmE. 18 cm

169. A 47-year-old male patient wasundergoing complex dental treatment.After anesthetization the patientreported dizziness, ringing in the ears,blackout, general weakness. Ps- 96bpm, AP- 90/60 mm Hg. The patientlost consciousness for 30 seconds. Whatcondition developed in the patient?

Page 27: Krok 2 - 2014 Question Paper (Stomatology)

Кrok 2 Stomatology (англомовнi студенти) 2014 рiк 27

A. SyncopeB. CollapseC. Anaphylactic shockD. Epileptic seizureE. Traumatic shock

170. A patient consulted a dentist aboutfabrication of a restorative crown forthe 36 tooth. During examination thedentist revealed a roundish, hard, pai-nless, lustrous ulcer on the left lateralsurface of tongue. What is the most li-kely diagnosis?

A. Syphilitic ulcerB. Decubital ulcerC. Tuberculous ulcerD. Vincent’s necrotizing ulcerativestomatitisE. Lichen planus

171. A 65-year-old male patientcomplains of a long-lasting pain in amandibular tooth on the right. Thepain increases on biting. The patienthas a history of this tooth treatment forcaries. Objectively: the cavity on themasticatory surface of the 36 tooth isfilled, percussion causes pain responce.Along the mucogingival junction in theprojection of the 36 tooth roots there isa fistula with purulent discharge. Whatis the most likely diagnosis?

A. Chronic granulating periodontitisB. Radicular cyst of the mandibleC. Exacerbation of chronic periodontitisD. Chronic fibrous periodontitisE. Chronic granulomatous periodontitis

172. Prior to installation of completeremovable dentures, it is necessaryto verify the occlusal relation withdifferent movements of the mandi-ble. What muscle is responsible fortransversal movements of the mandi-ble?

A. Outer (lateral) pterygopalatineB. Inner (medial) pterygopalatineC. TemporalD. MasticatoryE. Digastric

173. A 65-year-old male patientcomplains of crepitation and clicki-ng in both temporomandibular joi-nts; pain when moving his mandibleto the right; ear noise; dry mouth;

glossalgia. He has been using acomplete removable denture for themandible for 6 months. The patient hasno history of rheumatosis. Objectively:the lower third of the face is shortened,mental fold is strongly pronounced,mouth corners are downturned, thereare angular fissures and cracks. Palpati-on reveals crepitation during TMJmoving. What is the most likely di-agnosis?

A. Costen’s syndromeB. Temporomandibular arthritisC. Temporomandibular arthrosisD. Temporomandibular dislocationE. Temporomandibular osteoarthritis

174. A 76-year-old patient has a roundi-sh ulcer on the skin in the region of theright nasolabial sulcus. The ulcer flooris dense, rough, covered with yellowish-gray crusts, looks like a funnel. Theulcer edges are dense, crater-like. Theulcer grows in size and depth. What isthe provisional diagnosis?

A. BasaliomaB. Tuberculous ulcerC. Actinomycotic abscessD. Syphilitic ulcerE. Squamous cell skin cancer

175. A 59-year-old male patientpresented to a prosthetic dentistryclinic for the purpose of prosthetics.Immediately during the manipulati-ons the patient developed the followi-ng symptoms: weakness, prostration,skin pallor and cyanosis, clammy sweat,a decrease in systolic blood pressuredown to 80 mm Hg. What is the mostlikely diagnosis?

A. CollapseB. SyncopeC. ShockD. Kussmaul’s comaE. Myocardial infarction

176. A 40-year-old male patientcomplains of a long-existing fissure inthe central portion of the vermilionborder of his lower lip. The patienthas been a smoker since the age of16. Along the midline of the vermi-lion border of the lower lip there isa deep fissure up to 1,0 cm long wi-th the rolled edges. The fissure heals

Page 28: Krok 2 - 2014 Question Paper (Stomatology)

Кrok 2 Stomatology (англомовнi студенти) 2014 рiк 28

spontaneously, but recurs soon afterepithelialization. The patient was di-agnosed with chronic lower lip fissure.Conservative treatment proved to beineffective. Select the most efficientmethod of treatment:

A. Excision within apparently healthytissuesB. CryolysisC. DiathermocoagulationD. Excision within 0,5 cm area aroundthe formationE. Short-distance irradiation

177. A patient is scheduled for thedissection of an abscessed upper lipfuruncle. What kind of anesthesia isindicated for this intervention?

A. Bilateral infraorbitalB. Bilateral infraorbital and incisiveC. Bilateral infraorbital and palatinenerve blockD. Bilateral tuberal and palatine nerveblockE. Bilateral palatine nerve block

178. An accident victim with an injuryof face and the temporal region wasdiagnosed with a fracture of thezygomatic arch. Which processes of theskull bones are damaged?

A. Temporal process of the zygomaticbone and the zygomatic process of thetemporal boneB. Zygomatic process of the frontalbone and the zygomatic process of thetemporal boneC. Temporal process of the zygomaticbone and the zygomatic process of thefrontal boneD. Zygomatic process of the maxilla andthe zygomatic process of the temporalboneE. Zygomatic process of the maxillaand the zygomatic process of the frontalbone

179. With the purpose of the 46 toothextraction, a 28-year-old male pati-ent was given Weisbrem’s anesthesia.Specify the site of anesthetic injectionwith this anesthesia:

A. Mandibular torusB. Maxillary tuberC. Retromolar fossaD. Mandibular temporal crestE. Retromandibular fossa

180. A 7-year-old child with a markedbilateral swelling of the parotid regions,tenderness on palpation of the tragus,condylar and mastoid process, drynessof the oral mucosa was diagnosed withmumps of a mild severity grade. Selecta medical tactic with respect to thisseverity grade:

A. Outpatient treatment with isolationof the childB. Outpatient treatment without isolati-on of the childC. Hospitalization in children’sdepartment of oral and maxillofaci-al SurgeryD. The child is allowed to attend chi-ldren’s institutionsE. Hospitalization in the infectiousdiseases hospital

181. Examination of a 12-year-oldchild revealed a significant increasein the size of the jaws, presenceof gaps between the teeth, tongueenlargement, disproportionate skeletalgrowth. This pathology has beencaused by the dysfunction of thefollowing gland:

A. PituitaryB. ThyroidC. ParathyroidD. PancreasE. Sex

182. An 11-year-old boy had been di-agnosed with a 6 mm wide diastema oftype 2 (by Khoroshilkina F.I. classifi-cation). To normalize the incisor posi-tion, the Korkhaus appliance was used.What kind of movement is facilitatedby this appliance?

A. Bodily movementB. RotationC. IntrusionD. ExtrusionE. Torque

183. A 24-year-old male patientcomplains of spontaneous pain in the24 tooth, which arose suddenly and

Page 29: Krok 2 - 2014 Question Paper (Stomatology)

Кrok 2 Stomatology (англомовнi студенти) 2014 рiк 29

persists for about 15 minutes. Objecti-vely: the distal surface of the 24 toothexhibits a deep carious cavity withoverhanging walls. The cavity is fi-lled with light softened dentin andcommunicates with the tooth cavity.The cold stimulus causes acute, slowlyabating pain. Percussion causes no painresponse. Select the best method oftreatment:

A. Vital extirpationB. Vital amputationC. Biological methodD. Devital amputationE. Devital extirpation

184. A 9-year-old girl has had gingi-val hemorrhages, cracks in the mouthcorners for a month. She has a hi-story of frequent nosebleeds, rapid fati-gability. Objectively: the facial skinand oral mucosa are pale. In themouth corners there are cracks reachi-ng the vermilion border. Gingiva inthe region of the 11 and 12 teethis hyperemic and edematous, bleedswhen touched. DMF/dmf = 10. Back ofthe tongue is bright red, smooth, glossy.Submandibular lymph nodes are sli-ghtly enlarged, mobile, painless. Whatspecialist is it necessary to consult within the first place?

A. HematologistB. NeuropathistC. EndocrinologistD. Infectious disease specialistE. Gastroenterologist

185. A 25-year-old patient complains ofpain when biting on the 15 tooth. Thepain arose two days ago, has a constantaching nature and increased signifi-cantly over the last day. Objectively: thecrown of the 15 tooth is gray, the medialcontact surface exhibits a deep cariouscavity communicating with the toothcavity. Percussion causes acute pain, thegingival mucosa in the projection of the25 tooth root apex is hyperemic. Theregional lymph node is tender. Radi-ograph shows an ill-defined zone ofperiapical bone destruction. What isthe most likely diagnosis?

A. Exacerbation of chronic periodonti-tisB. Acute serous periodontitisC. Acute suppurative periodontitisD. Chronic granulating periodontitisE. Acute serous periodontitis, intoxi-cation stage

186. A 5-month-old infant has acutehematogenous osteomyelitis of themaxilla, there are fistulae with purulentexudate at the medial angle of the ri-ght eye. Specify the probable long-termeffects of the disease:

A. Bony ankylosis of the right TMJB. Chronic sinusitisC. Chronic rhinitisD. MacrognathiaE. Macrogenia

187. A 16-year-old boy has been di-agnosed with a fracture of the leftzygomatic bone with a fragment di-splacement. Select the most efficientmethod of treatment:

A. Open reposition and osteosynthesiswith metal platesB. Osteosynthesis with metal wiresC. Any of these methods is applied inchildren and adolescentsD. Closed repositionE. Bone suture

188. A 5-year-old girl complains of painin the mouth corners occurring whenopening the mouth. She has a history ofacute bronchitis and prolonged antibi-otic therapy. Objectively: the vermilionborder of lips is dry, the mouth cornersare downturned. In the mouth cornersthere are cracks covered with a whi-te plaque and surrounded by a slightlyhyperemic elevation, tender on palpati-on and mouth opening. Oral mucosaexhibits no pathological alterations,dmf - 2, hygiene index - 2,0. What isthe most likely diagnosis?

A. Mycotic cheilitisB. Streptococcal cheilitisC. Meteorological cheilitisD. Glandular cheilitisE. Exfoliative cheilitis

189. A female patient complains of amoderate gum pain, tooth sensitivityto the thermal stimuli. Objectively: the

Page 30: Krok 2 - 2014 Question Paper (Stomatology)

Кrok 2 Stomatology (англомовнi студенти) 2014 рiк 30

gingiva is pale, dense, there is gingi-val retraction in the region of the frontmandibular teeth. Radiograph showsthe resorption of the interalveolarsepta to 1/3 of the root length. Whatis the most likely diagnosis?

A. Periodontosis grade IB. Periodontosis grade IIC. Periodontosis initial stageD. Catarrhal gingivitisE. Localized periodontitis

190. A 60-year-old patient needs apartial removable denture for the maxi-lla. The centric jaw relation has beendetermined. What is the next stage ofprosthetics?

A. Checking the wax denture constructi-onB. Try-in of the dentureC. Fabrication of plaster modelsD. Relining the dentureE. Taking impressions

191. A 40-year-old patient complai-ns of clicking in the region of his ri-ght TMJ projection which occurs inthe final phase of mouth opening.The sounds in the joints arose afterprosthetic procedures. Mouth openingis not restricted, the opening trajectoryis straight, the opening amplitude is 5cm. What is the most likely diagnosis?

A. Subluxation of mandibleB. Luxation of mandibleC. Anterior dislocation of the articulardiscD. Posterior dislocation of the articulardiscE. Perforation of the meniscus

192. A mother consulted a denti-st about the lip lesions in her child.Throughout the 1-2 year of life, the chi-ld had diathesis of moderate severitywith affection of the face skin. Objecti-vely: the lips are somewhat edematous,there are dry crusts on the vermilionborder and the lip skin, in the mouthcorners there are cracks and radialscars. The eyelids are edematous wi-th hyperemic edges. What is the mostlikely diagnosis?

A. Eczematous cheilitisB. Exfoliative cheilitisC. Glandular cheilitisD. Meteorological cheilitisE. Chronic fissure of lips

193. Examination of a 7-year-old chi-ld revealed that all temporary maxi-llary molars were extracted. Mandi-bular incisors were in contact with thepalatal mucosa. What is the optimaldoctor’s tactics?

A. Fabrication of a removable laminarclaspless denture for the maxillary teethrestorationB. Fabrication of an orthodontic devicefor the treatment of deep overbiteC. Fabrication of a removable denturewith clasp fixation for the maxillaryteeth restorationD. Check-ups every six months until theeruption of permanent teethE. Check-ups once a year until theeruption of permanent teeth

194. A 34-year-old dental patient hasan indication for the extraction of the38 tooth for chronic fibrous periodonti-tis. The dental surgeon used torusal(Weisbrem’s) anesthesia. What nervesare blocked by this anesthesia?

A. Superior alveolar, lingual, buccalB. Lingual, buccalC. Superior alveolar, buccalD. Superior alveolar, lingualE. Lingual, buccal, mental

195. A 40-year-old patient has anindication for the extraction ofcentral maxillary incisors. What nervestructures are blocked by the incisoranesthesia?

A. Nasopalatine nerveB. Anterior superior alveolar branch ofthe infraorbital nerveC. Middle superior alveolar branch ofthe infraorbital nerveD. Posterior superior alveolar branch ofthe infraorbital nerveE. Dental nerve plexus

196. A 40-year-old male patientcomplains of having experiencedseasonal pain (in spring and summer),and a burning sensation in the lips for3 years. Examination reveals edema

Page 31: Krok 2 - 2014 Question Paper (Stomatology)

Кrok 2 Stomatology (англомовнi студенти) 2014 рiк 31

and hyperemia of the vermilion borderof the lower lip, numerous erosionssized 2-3 mm, crusts and cracks inthe mouth corners. Palpation causesmoderate pain, regional lymph nodesare not enlarged, the perioral skin isnot changed. What is most likely provi-sional diagnosis?

A. Actinic cheilitisB. Exfoliative cheilitis, edematous formC. Eczematous cheilitisD. Lichen planus, erosive formE. Mycotic angular cheilitis

197. A 25-year-old male patient lives ina locality where fluoride concentrati-on in drinking water is 5,5 mg/l. Toothcrowns are deformed due to destructi-on and wear of hard tissues. In the22, 23, 32 teeth the enamel is chipped.What form of fluorosis is observed inthis patient?

A. DestructiveB. StreakedC. SpottyD. Chalky and speckledE. Erosive

198. Prior to the preparation of a toothfor a porcelain-fused-to-metal crown,a dentist anaesthetized a patient wi-th a history of bronchial asthma wi-th 2% solution of novocaine. A fewminutes later, the patient complai-ned of shortness of breath (moredifficult expiration), there appearedacrocyanosis, swollen neck veins, Ps-100 bpm, rhythmic, AP- 180/110 mmHg. Percussion revealed box soundover lungs. What is the most likely di-agnosis?

A. Attack of bronchial asthmaB. Coronary artery diseaseC. Pulmonary artery thrombosisD. BronchiectasisE. Collapse

199. A 12-year-old girl complains ofacute pain in the left mandibular regi-on, general weakness, fever up to39oC, difficult swallowing. Objectively:the face is asymmetric, the tongue isplaqued, halitosis is present. Percussi-on reveals signs of acute periodontitisof the 34, 36 teeth, as well as mobili-ty of these teeth. Gums are hyperemic,edematous, there is infiltration on bothsides of the alveolar process. Regi-onal lymph nodes are enlarged, tenderon palpation. Vincent’s symptom ispresent. Blood test results: leukocytosiswith a left shift; urine contains tracesof protein. What is the most likely di-agnosis?

A. Acute odontogenic osteomyelitis ofthe mandibleB. Acute odontogenic purulent abscessC. Exacerbation of chronic periodontitisD. Acute odontogenic purulentlymphadenitisE. Odontogenic submandibular abscess

200. Parents of a 3-year-old child tookthe child to a pedodontist for complexdental care. The child has cerebralpalsy. Objectively: there is multi-ple dental caries, gingival mucosa ishyperemic, oral hygiene is inadequate.What method of dental care may beappropriate in this case?

A. Under general anesthesiaB. Under local injection anesthesiaC. Under local application anesthesiaD. Without anesthesiaE. Treatment after administration ofsedatives